Maximum value of $x$ when equality is given











up vote
4
down vote

favorite












$$ x + y = sqrt{x} + sqrt{y} $$
Find maximum value of $x$. $x$ and $y$ are reals.










share|cite|improve this question
























  • "Find maximum value...of $x$"? What does this mean?
    – DonAntonio
    Nov 17 at 20:17












  • it means most probably to find max value of x satisfying the equation
    – maveric
    Nov 17 at 20:18






  • 3




    Ok, I think I understand now. When $;x,yinBbb N;$ (since they obviously must be non-negative), what is the maximum value for some $;x;$ fulfilling that no matter what $;y;$ we take...A little odd question.
    – DonAntonio
    Nov 17 at 20:20










  • The question makes no much sense, as @DonAntonio suggests.
    – Rebellos
    Nov 17 at 20:22










  • Solve as quadric with respect to $sqrt{x}$ and then maximize the root value + the condition on integers. Would that not work?
    – Makina
    Nov 17 at 20:23















up vote
4
down vote

favorite












$$ x + y = sqrt{x} + sqrt{y} $$
Find maximum value of $x$. $x$ and $y$ are reals.










share|cite|improve this question
























  • "Find maximum value...of $x$"? What does this mean?
    – DonAntonio
    Nov 17 at 20:17












  • it means most probably to find max value of x satisfying the equation
    – maveric
    Nov 17 at 20:18






  • 3




    Ok, I think I understand now. When $;x,yinBbb N;$ (since they obviously must be non-negative), what is the maximum value for some $;x;$ fulfilling that no matter what $;y;$ we take...A little odd question.
    – DonAntonio
    Nov 17 at 20:20










  • The question makes no much sense, as @DonAntonio suggests.
    – Rebellos
    Nov 17 at 20:22










  • Solve as quadric with respect to $sqrt{x}$ and then maximize the root value + the condition on integers. Would that not work?
    – Makina
    Nov 17 at 20:23













up vote
4
down vote

favorite









up vote
4
down vote

favorite











$$ x + y = sqrt{x} + sqrt{y} $$
Find maximum value of $x$. $x$ and $y$ are reals.










share|cite|improve this question















$$ x + y = sqrt{x} + sqrt{y} $$
Find maximum value of $x$. $x$ and $y$ are reals.







algebra-precalculus






share|cite|improve this question















share|cite|improve this question













share|cite|improve this question




share|cite|improve this question








edited 2 days ago









iBug

1427




1427










asked Nov 17 at 20:13









maveric

59511




59511












  • "Find maximum value...of $x$"? What does this mean?
    – DonAntonio
    Nov 17 at 20:17












  • it means most probably to find max value of x satisfying the equation
    – maveric
    Nov 17 at 20:18






  • 3




    Ok, I think I understand now. When $;x,yinBbb N;$ (since they obviously must be non-negative), what is the maximum value for some $;x;$ fulfilling that no matter what $;y;$ we take...A little odd question.
    – DonAntonio
    Nov 17 at 20:20










  • The question makes no much sense, as @DonAntonio suggests.
    – Rebellos
    Nov 17 at 20:22










  • Solve as quadric with respect to $sqrt{x}$ and then maximize the root value + the condition on integers. Would that not work?
    – Makina
    Nov 17 at 20:23


















  • "Find maximum value...of $x$"? What does this mean?
    – DonAntonio
    Nov 17 at 20:17












  • it means most probably to find max value of x satisfying the equation
    – maveric
    Nov 17 at 20:18






  • 3




    Ok, I think I understand now. When $;x,yinBbb N;$ (since they obviously must be non-negative), what is the maximum value for some $;x;$ fulfilling that no matter what $;y;$ we take...A little odd question.
    – DonAntonio
    Nov 17 at 20:20










  • The question makes no much sense, as @DonAntonio suggests.
    – Rebellos
    Nov 17 at 20:22










  • Solve as quadric with respect to $sqrt{x}$ and then maximize the root value + the condition on integers. Would that not work?
    – Makina
    Nov 17 at 20:23
















"Find maximum value...of $x$"? What does this mean?
– DonAntonio
Nov 17 at 20:17






"Find maximum value...of $x$"? What does this mean?
– DonAntonio
Nov 17 at 20:17














it means most probably to find max value of x satisfying the equation
– maveric
Nov 17 at 20:18




it means most probably to find max value of x satisfying the equation
– maveric
Nov 17 at 20:18




3




3




Ok, I think I understand now. When $;x,yinBbb N;$ (since they obviously must be non-negative), what is the maximum value for some $;x;$ fulfilling that no matter what $;y;$ we take...A little odd question.
– DonAntonio
Nov 17 at 20:20




Ok, I think I understand now. When $;x,yinBbb N;$ (since they obviously must be non-negative), what is the maximum value for some $;x;$ fulfilling that no matter what $;y;$ we take...A little odd question.
– DonAntonio
Nov 17 at 20:20












The question makes no much sense, as @DonAntonio suggests.
– Rebellos
Nov 17 at 20:22




The question makes no much sense, as @DonAntonio suggests.
– Rebellos
Nov 17 at 20:22












Solve as quadric with respect to $sqrt{x}$ and then maximize the root value + the condition on integers. Would that not work?
– Makina
Nov 17 at 20:23




Solve as quadric with respect to $sqrt{x}$ and then maximize the root value + the condition on integers. Would that not work?
– Makina
Nov 17 at 20:23










7 Answers
7






active

oldest

votes

















up vote
4
down vote













Try this method of completing the square.



Let $a=sqrt x$ and $b=sqrt y$ so that $a^2+b^2=a+b$ and $4a^2-4a+1+4b^2-4b+1=2$ ie $(2a-1)^2+(2b-1)^2=2$






share|cite|improve this answer




























    up vote
    4
    down vote













    enter image description here



    If you set $f(x)=x-sqrt{x}$ then you want to solve $f(x)=-f(y)$



    The maximum $x$ is reached for the line in green intersecting both curves $f$ and $-f$ at the maximal possible altitude.



    So we have to solve $f'(y)=0iff1-dfrac1{2sqrt{y}}=0iff y=frac 14$



    The corresponding $x$ verifies $f(x)=-f(y)=dfrac 14$ and this is just a quadratic polynomial to solve to find the $x$ value.



    The maximum is then $x=frac{3+2sqrt{2}}4approx 1.4571$






    share|cite|improve this answer



















    • 1




      dont understant what you trying to convey?
      – maveric
      Nov 17 at 20:50


















    up vote
    3
    down vote













    Obviously $x,yge0$. We can therefore write $x=r^2cos^2theta$ and $y=r^2sin^2theta$. Then the equation becomes $$r^2(cos^2theta+sin^2theta)=r(costheta+sintheta)$$
    with $theta in [0,fracpi2]$ and $rgt 0$. Dividing by $r$ and multiplying by $costheta$ we get $$sqrt x=(costheta+sintheta)costheta=cos^2theta+sinthetacostheta$$
    Now find the maximum of the expression on the right. Use formula for the sine and cosine for double angle.






    share|cite|improve this answer




























      up vote
      2
      down vote













      Since $x+ y = sqrt{x} + sqrt{y}$ is symmetric, WLOG, we can consider $xge yge 0$ or $y=ax, 0le ale 1$. Then:
      $$x+ ax = sqrt{x} + sqrt{ax} Rightarrow color{red}{sqrt{x}}=frac{1+sqrt{a}}{1+a} to text{max s.t. $0le ale 1$}\
      f'(a)=left(frac{1+sqrt{a}}{1+a}right)'=0 Rightarrow frac{frac1{2sqrt{a}}(1+a)-(1+sqrt{a})}{(1+a)^2}=0 Rightarrow \
      a+2sqrt{a}-1=0 Rightarrow a=3-2sqrt{2}approx 0.17.$$

      Note that the function $f(a)$ is continuous at $0le ale 1$ and $f(0)=f(1)=1$ and $f(3-2sqrt{2})=frac12(1+sqrt{2})approx 1.207$. Hence it must be the maximum point (see Desmos graph).



      So, the maximum value of $x$ is:
      $$color{red}x=left(frac{1+sqrt{3-2sqrt{2}}}{1+(3-2sqrt{2})}right)^2=frac14left(1+sqrt{2}right)^2approx 1.457.$$






      share|cite|improve this answer























      • I don't think that's right. See zwim's answer, or just graph the curve in question to see that the right value should be around $1.46$.
        – YiFan
        Nov 18 at 6:25






      • 2




        $1.207^2approx 1.457$ which is the right answer, so the method is fine. There is an error in the last calculation of $x$.
        – zwim
        Nov 18 at 6:32






      • 1




        @zwim, thank you for pointing to the error. Fixed it.
        – farruhota
        Nov 18 at 6:43










      • +1. Nice method.
        – Taladris
        2 days ago


















      up vote
      1
      down vote













      Observe that



      $$x+y=sqrt x+sqrt yiff sqrt xleft(sqrt x-1right)=sqrt yleft(1-sqrt yright)$$



      Now, for any values $;x,y>1;$ , the left side above will be positive whereas the right side will be negative (can you see why?), thus we must restrict the possible natural values...Can you take it from here?






      share|cite|improve this answer





















      • sorry the question is edited. it was real.
        – maveric
        Nov 17 at 20:25










      • can we say then 1?
        – maveric
        Nov 17 at 20:25










      • The above solves completely the first, original question. Now that it has changed to "real" it at least can help you to realize that $;y;$ cannot be greater than $;1;$ and $;x;$ smaller than one, for example...
        – DonAntonio
        Nov 17 at 20:39


















      up vote
      0
      down vote













      Let $f(x)=x-sqrt x$. The question becomes trying to find two real values $x,y$ so that $f(x)=-f(y)$, then maximising one of these values without regard for what happens to the other. After a bit of thinking analysing derivatives, it becomes clear that we want to minimise $f(x)$ so that $f(y)$, and hence $y$, can be maximised. So just set $f'(x)=0$ to find the local minimum value to be $-1/4$, and hence the maximum value of $y$ satisfies $f(y)=1/4$. This is a simple quadratic equation, solve, and you're done!






      share|cite|improve this answer




























        up vote
        0
        down vote













        There are some caveats (we need to assume the existence of a maximum for example) but Lagrange's multipliers method work well here to find the value of the maximum:



        Let $f(x,y)=x$ and $g(x,y)=x-sqrt{x}+y-sqrt{y}$. We want to find the maximum value of $f$ under constraint $g(x,y)=0$.



        We have $nabla f=langle 1,0 rangle$ and $nabla g = langle 1-frac{1}{2sqrt{x}},1-frac{1}{2sqrt{y}} rangle$. At a maximum, we have $nabla g=lambda nabla f$ for some real number $lambda$. This implies that $1-frac{1}{2sqrt{y}}=0$, that is $y=frac{1}{4}$.



        Using the constraint, we have $x-sqrt{x}-frac{1}{4}=0$ which is equivalent to $a^2-a-frac{1}{4}$ and $sqrt{x}=ageqslant 0$. We obtain $a=frac{1+sqrt{2}}{2}$, so $x=frac{(1+sqrt{2})^2}{4}=frac{3+2sqrt{2}}{4}$.





        (sloppy) proof of the existence of the global maximum value: The constraint $x+y=sqrt{x}+sqrt{y}$ implies that $xgeqslant 0$ and $ygeqslant 0$. Moreover, if $x$ or $y$ is large enough (x>R or y>R for some $R$ that I am too lazy to determine explicitly :p), $x+y>sqrt{x}+sqrt{y}$, so we can assume that $x,y in [0,R]$.



        Now, the function $f(x,y)=x$ is continuous and the set $S=left{ (x,y)in [0,R]^2; middle|; g(x,y)=0; right}$ is compact (closed and bounded in ${mathbb R}^2$) so, by the Extreme Value Theorem, $f$ attains its global extreme values on $S$. The global minimum value is obviously obtained at $(0,0)$, while the other one is located by Lagrange's method.






        share|cite|improve this answer





















          Your Answer





          StackExchange.ifUsing("editor", function () {
          return StackExchange.using("mathjaxEditing", function () {
          StackExchange.MarkdownEditor.creationCallbacks.add(function (editor, postfix) {
          StackExchange.mathjaxEditing.prepareWmdForMathJax(editor, postfix, [["$", "$"], ["\\(","\\)"]]);
          });
          });
          }, "mathjax-editing");

          StackExchange.ready(function() {
          var channelOptions = {
          tags: "".split(" "),
          id: "69"
          };
          initTagRenderer("".split(" "), "".split(" "), channelOptions);

          StackExchange.using("externalEditor", function() {
          // Have to fire editor after snippets, if snippets enabled
          if (StackExchange.settings.snippets.snippetsEnabled) {
          StackExchange.using("snippets", function() {
          createEditor();
          });
          }
          else {
          createEditor();
          }
          });

          function createEditor() {
          StackExchange.prepareEditor({
          heartbeatType: 'answer',
          convertImagesToLinks: true,
          noModals: true,
          showLowRepImageUploadWarning: true,
          reputationToPostImages: 10,
          bindNavPrevention: true,
          postfix: "",
          imageUploader: {
          brandingHtml: "Powered by u003ca class="icon-imgur-white" href="https://imgur.com/"u003eu003c/au003e",
          contentPolicyHtml: "User contributions licensed under u003ca href="https://creativecommons.org/licenses/by-sa/3.0/"u003ecc by-sa 3.0 with attribution requiredu003c/au003e u003ca href="https://stackoverflow.com/legal/content-policy"u003e(content policy)u003c/au003e",
          allowUrls: true
          },
          noCode: true, onDemand: true,
          discardSelector: ".discard-answer"
          ,immediatelyShowMarkdownHelp:true
          });


          }
          });














           

          draft saved


          draft discarded


















          StackExchange.ready(
          function () {
          StackExchange.openid.initPostLogin('.new-post-login', 'https%3a%2f%2fmath.stackexchange.com%2fquestions%2f3002761%2fmaximum-value-of-x-when-equality-is-given%23new-answer', 'question_page');
          }
          );

          Post as a guest















          Required, but never shown

























          7 Answers
          7






          active

          oldest

          votes








          7 Answers
          7






          active

          oldest

          votes









          active

          oldest

          votes






          active

          oldest

          votes








          up vote
          4
          down vote













          Try this method of completing the square.



          Let $a=sqrt x$ and $b=sqrt y$ so that $a^2+b^2=a+b$ and $4a^2-4a+1+4b^2-4b+1=2$ ie $(2a-1)^2+(2b-1)^2=2$






          share|cite|improve this answer

























            up vote
            4
            down vote













            Try this method of completing the square.



            Let $a=sqrt x$ and $b=sqrt y$ so that $a^2+b^2=a+b$ and $4a^2-4a+1+4b^2-4b+1=2$ ie $(2a-1)^2+(2b-1)^2=2$






            share|cite|improve this answer























              up vote
              4
              down vote










              up vote
              4
              down vote









              Try this method of completing the square.



              Let $a=sqrt x$ and $b=sqrt y$ so that $a^2+b^2=a+b$ and $4a^2-4a+1+4b^2-4b+1=2$ ie $(2a-1)^2+(2b-1)^2=2$






              share|cite|improve this answer












              Try this method of completing the square.



              Let $a=sqrt x$ and $b=sqrt y$ so that $a^2+b^2=a+b$ and $4a^2-4a+1+4b^2-4b+1=2$ ie $(2a-1)^2+(2b-1)^2=2$







              share|cite|improve this answer












              share|cite|improve this answer



              share|cite|improve this answer










              answered Nov 17 at 20:25









              Mark Bennet

              79.6k978177




              79.6k978177






















                  up vote
                  4
                  down vote













                  enter image description here



                  If you set $f(x)=x-sqrt{x}$ then you want to solve $f(x)=-f(y)$



                  The maximum $x$ is reached for the line in green intersecting both curves $f$ and $-f$ at the maximal possible altitude.



                  So we have to solve $f'(y)=0iff1-dfrac1{2sqrt{y}}=0iff y=frac 14$



                  The corresponding $x$ verifies $f(x)=-f(y)=dfrac 14$ and this is just a quadratic polynomial to solve to find the $x$ value.



                  The maximum is then $x=frac{3+2sqrt{2}}4approx 1.4571$






                  share|cite|improve this answer



















                  • 1




                    dont understant what you trying to convey?
                    – maveric
                    Nov 17 at 20:50















                  up vote
                  4
                  down vote













                  enter image description here



                  If you set $f(x)=x-sqrt{x}$ then you want to solve $f(x)=-f(y)$



                  The maximum $x$ is reached for the line in green intersecting both curves $f$ and $-f$ at the maximal possible altitude.



                  So we have to solve $f'(y)=0iff1-dfrac1{2sqrt{y}}=0iff y=frac 14$



                  The corresponding $x$ verifies $f(x)=-f(y)=dfrac 14$ and this is just a quadratic polynomial to solve to find the $x$ value.



                  The maximum is then $x=frac{3+2sqrt{2}}4approx 1.4571$






                  share|cite|improve this answer



















                  • 1




                    dont understant what you trying to convey?
                    – maveric
                    Nov 17 at 20:50













                  up vote
                  4
                  down vote










                  up vote
                  4
                  down vote









                  enter image description here



                  If you set $f(x)=x-sqrt{x}$ then you want to solve $f(x)=-f(y)$



                  The maximum $x$ is reached for the line in green intersecting both curves $f$ and $-f$ at the maximal possible altitude.



                  So we have to solve $f'(y)=0iff1-dfrac1{2sqrt{y}}=0iff y=frac 14$



                  The corresponding $x$ verifies $f(x)=-f(y)=dfrac 14$ and this is just a quadratic polynomial to solve to find the $x$ value.



                  The maximum is then $x=frac{3+2sqrt{2}}4approx 1.4571$






                  share|cite|improve this answer














                  enter image description here



                  If you set $f(x)=x-sqrt{x}$ then you want to solve $f(x)=-f(y)$



                  The maximum $x$ is reached for the line in green intersecting both curves $f$ and $-f$ at the maximal possible altitude.



                  So we have to solve $f'(y)=0iff1-dfrac1{2sqrt{y}}=0iff y=frac 14$



                  The corresponding $x$ verifies $f(x)=-f(y)=dfrac 14$ and this is just a quadratic polynomial to solve to find the $x$ value.



                  The maximum is then $x=frac{3+2sqrt{2}}4approx 1.4571$







                  share|cite|improve this answer














                  share|cite|improve this answer



                  share|cite|improve this answer








                  edited Nov 18 at 6:21

























                  answered Nov 17 at 20:47









                  zwim

                  11.2k628




                  11.2k628








                  • 1




                    dont understant what you trying to convey?
                    – maveric
                    Nov 17 at 20:50














                  • 1




                    dont understant what you trying to convey?
                    – maveric
                    Nov 17 at 20:50








                  1




                  1




                  dont understant what you trying to convey?
                  – maveric
                  Nov 17 at 20:50




                  dont understant what you trying to convey?
                  – maveric
                  Nov 17 at 20:50










                  up vote
                  3
                  down vote













                  Obviously $x,yge0$. We can therefore write $x=r^2cos^2theta$ and $y=r^2sin^2theta$. Then the equation becomes $$r^2(cos^2theta+sin^2theta)=r(costheta+sintheta)$$
                  with $theta in [0,fracpi2]$ and $rgt 0$. Dividing by $r$ and multiplying by $costheta$ we get $$sqrt x=(costheta+sintheta)costheta=cos^2theta+sinthetacostheta$$
                  Now find the maximum of the expression on the right. Use formula for the sine and cosine for double angle.






                  share|cite|improve this answer

























                    up vote
                    3
                    down vote













                    Obviously $x,yge0$. We can therefore write $x=r^2cos^2theta$ and $y=r^2sin^2theta$. Then the equation becomes $$r^2(cos^2theta+sin^2theta)=r(costheta+sintheta)$$
                    with $theta in [0,fracpi2]$ and $rgt 0$. Dividing by $r$ and multiplying by $costheta$ we get $$sqrt x=(costheta+sintheta)costheta=cos^2theta+sinthetacostheta$$
                    Now find the maximum of the expression on the right. Use formula for the sine and cosine for double angle.






                    share|cite|improve this answer























                      up vote
                      3
                      down vote










                      up vote
                      3
                      down vote









                      Obviously $x,yge0$. We can therefore write $x=r^2cos^2theta$ and $y=r^2sin^2theta$. Then the equation becomes $$r^2(cos^2theta+sin^2theta)=r(costheta+sintheta)$$
                      with $theta in [0,fracpi2]$ and $rgt 0$. Dividing by $r$ and multiplying by $costheta$ we get $$sqrt x=(costheta+sintheta)costheta=cos^2theta+sinthetacostheta$$
                      Now find the maximum of the expression on the right. Use formula for the sine and cosine for double angle.






                      share|cite|improve this answer












                      Obviously $x,yge0$. We can therefore write $x=r^2cos^2theta$ and $y=r^2sin^2theta$. Then the equation becomes $$r^2(cos^2theta+sin^2theta)=r(costheta+sintheta)$$
                      with $theta in [0,fracpi2]$ and $rgt 0$. Dividing by $r$ and multiplying by $costheta$ we get $$sqrt x=(costheta+sintheta)costheta=cos^2theta+sinthetacostheta$$
                      Now find the maximum of the expression on the right. Use formula for the sine and cosine for double angle.







                      share|cite|improve this answer












                      share|cite|improve this answer



                      share|cite|improve this answer










                      answered Nov 17 at 20:47









                      Andrei

                      9,68621024




                      9,68621024






















                          up vote
                          2
                          down vote













                          Since $x+ y = sqrt{x} + sqrt{y}$ is symmetric, WLOG, we can consider $xge yge 0$ or $y=ax, 0le ale 1$. Then:
                          $$x+ ax = sqrt{x} + sqrt{ax} Rightarrow color{red}{sqrt{x}}=frac{1+sqrt{a}}{1+a} to text{max s.t. $0le ale 1$}\
                          f'(a)=left(frac{1+sqrt{a}}{1+a}right)'=0 Rightarrow frac{frac1{2sqrt{a}}(1+a)-(1+sqrt{a})}{(1+a)^2}=0 Rightarrow \
                          a+2sqrt{a}-1=0 Rightarrow a=3-2sqrt{2}approx 0.17.$$

                          Note that the function $f(a)$ is continuous at $0le ale 1$ and $f(0)=f(1)=1$ and $f(3-2sqrt{2})=frac12(1+sqrt{2})approx 1.207$. Hence it must be the maximum point (see Desmos graph).



                          So, the maximum value of $x$ is:
                          $$color{red}x=left(frac{1+sqrt{3-2sqrt{2}}}{1+(3-2sqrt{2})}right)^2=frac14left(1+sqrt{2}right)^2approx 1.457.$$






                          share|cite|improve this answer























                          • I don't think that's right. See zwim's answer, or just graph the curve in question to see that the right value should be around $1.46$.
                            – YiFan
                            Nov 18 at 6:25






                          • 2




                            $1.207^2approx 1.457$ which is the right answer, so the method is fine. There is an error in the last calculation of $x$.
                            – zwim
                            Nov 18 at 6:32






                          • 1




                            @zwim, thank you for pointing to the error. Fixed it.
                            – farruhota
                            Nov 18 at 6:43










                          • +1. Nice method.
                            – Taladris
                            2 days ago















                          up vote
                          2
                          down vote













                          Since $x+ y = sqrt{x} + sqrt{y}$ is symmetric, WLOG, we can consider $xge yge 0$ or $y=ax, 0le ale 1$. Then:
                          $$x+ ax = sqrt{x} + sqrt{ax} Rightarrow color{red}{sqrt{x}}=frac{1+sqrt{a}}{1+a} to text{max s.t. $0le ale 1$}\
                          f'(a)=left(frac{1+sqrt{a}}{1+a}right)'=0 Rightarrow frac{frac1{2sqrt{a}}(1+a)-(1+sqrt{a})}{(1+a)^2}=0 Rightarrow \
                          a+2sqrt{a}-1=0 Rightarrow a=3-2sqrt{2}approx 0.17.$$

                          Note that the function $f(a)$ is continuous at $0le ale 1$ and $f(0)=f(1)=1$ and $f(3-2sqrt{2})=frac12(1+sqrt{2})approx 1.207$. Hence it must be the maximum point (see Desmos graph).



                          So, the maximum value of $x$ is:
                          $$color{red}x=left(frac{1+sqrt{3-2sqrt{2}}}{1+(3-2sqrt{2})}right)^2=frac14left(1+sqrt{2}right)^2approx 1.457.$$






                          share|cite|improve this answer























                          • I don't think that's right. See zwim's answer, or just graph the curve in question to see that the right value should be around $1.46$.
                            – YiFan
                            Nov 18 at 6:25






                          • 2




                            $1.207^2approx 1.457$ which is the right answer, so the method is fine. There is an error in the last calculation of $x$.
                            – zwim
                            Nov 18 at 6:32






                          • 1




                            @zwim, thank you for pointing to the error. Fixed it.
                            – farruhota
                            Nov 18 at 6:43










                          • +1. Nice method.
                            – Taladris
                            2 days ago













                          up vote
                          2
                          down vote










                          up vote
                          2
                          down vote









                          Since $x+ y = sqrt{x} + sqrt{y}$ is symmetric, WLOG, we can consider $xge yge 0$ or $y=ax, 0le ale 1$. Then:
                          $$x+ ax = sqrt{x} + sqrt{ax} Rightarrow color{red}{sqrt{x}}=frac{1+sqrt{a}}{1+a} to text{max s.t. $0le ale 1$}\
                          f'(a)=left(frac{1+sqrt{a}}{1+a}right)'=0 Rightarrow frac{frac1{2sqrt{a}}(1+a)-(1+sqrt{a})}{(1+a)^2}=0 Rightarrow \
                          a+2sqrt{a}-1=0 Rightarrow a=3-2sqrt{2}approx 0.17.$$

                          Note that the function $f(a)$ is continuous at $0le ale 1$ and $f(0)=f(1)=1$ and $f(3-2sqrt{2})=frac12(1+sqrt{2})approx 1.207$. Hence it must be the maximum point (see Desmos graph).



                          So, the maximum value of $x$ is:
                          $$color{red}x=left(frac{1+sqrt{3-2sqrt{2}}}{1+(3-2sqrt{2})}right)^2=frac14left(1+sqrt{2}right)^2approx 1.457.$$






                          share|cite|improve this answer














                          Since $x+ y = sqrt{x} + sqrt{y}$ is symmetric, WLOG, we can consider $xge yge 0$ or $y=ax, 0le ale 1$. Then:
                          $$x+ ax = sqrt{x} + sqrt{ax} Rightarrow color{red}{sqrt{x}}=frac{1+sqrt{a}}{1+a} to text{max s.t. $0le ale 1$}\
                          f'(a)=left(frac{1+sqrt{a}}{1+a}right)'=0 Rightarrow frac{frac1{2sqrt{a}}(1+a)-(1+sqrt{a})}{(1+a)^2}=0 Rightarrow \
                          a+2sqrt{a}-1=0 Rightarrow a=3-2sqrt{2}approx 0.17.$$

                          Note that the function $f(a)$ is continuous at $0le ale 1$ and $f(0)=f(1)=1$ and $f(3-2sqrt{2})=frac12(1+sqrt{2})approx 1.207$. Hence it must be the maximum point (see Desmos graph).



                          So, the maximum value of $x$ is:
                          $$color{red}x=left(frac{1+sqrt{3-2sqrt{2}}}{1+(3-2sqrt{2})}right)^2=frac14left(1+sqrt{2}right)^2approx 1.457.$$







                          share|cite|improve this answer














                          share|cite|improve this answer



                          share|cite|improve this answer








                          edited Nov 18 at 6:39

























                          answered Nov 18 at 5:32









                          farruhota

                          17.6k2736




                          17.6k2736












                          • I don't think that's right. See zwim's answer, or just graph the curve in question to see that the right value should be around $1.46$.
                            – YiFan
                            Nov 18 at 6:25






                          • 2




                            $1.207^2approx 1.457$ which is the right answer, so the method is fine. There is an error in the last calculation of $x$.
                            – zwim
                            Nov 18 at 6:32






                          • 1




                            @zwim, thank you for pointing to the error. Fixed it.
                            – farruhota
                            Nov 18 at 6:43










                          • +1. Nice method.
                            – Taladris
                            2 days ago


















                          • I don't think that's right. See zwim's answer, or just graph the curve in question to see that the right value should be around $1.46$.
                            – YiFan
                            Nov 18 at 6:25






                          • 2




                            $1.207^2approx 1.457$ which is the right answer, so the method is fine. There is an error in the last calculation of $x$.
                            – zwim
                            Nov 18 at 6:32






                          • 1




                            @zwim, thank you for pointing to the error. Fixed it.
                            – farruhota
                            Nov 18 at 6:43










                          • +1. Nice method.
                            – Taladris
                            2 days ago
















                          I don't think that's right. See zwim's answer, or just graph the curve in question to see that the right value should be around $1.46$.
                          – YiFan
                          Nov 18 at 6:25




                          I don't think that's right. See zwim's answer, or just graph the curve in question to see that the right value should be around $1.46$.
                          – YiFan
                          Nov 18 at 6:25




                          2




                          2




                          $1.207^2approx 1.457$ which is the right answer, so the method is fine. There is an error in the last calculation of $x$.
                          – zwim
                          Nov 18 at 6:32




                          $1.207^2approx 1.457$ which is the right answer, so the method is fine. There is an error in the last calculation of $x$.
                          – zwim
                          Nov 18 at 6:32




                          1




                          1




                          @zwim, thank you for pointing to the error. Fixed it.
                          – farruhota
                          Nov 18 at 6:43




                          @zwim, thank you for pointing to the error. Fixed it.
                          – farruhota
                          Nov 18 at 6:43












                          +1. Nice method.
                          – Taladris
                          2 days ago




                          +1. Nice method.
                          – Taladris
                          2 days ago










                          up vote
                          1
                          down vote













                          Observe that



                          $$x+y=sqrt x+sqrt yiff sqrt xleft(sqrt x-1right)=sqrt yleft(1-sqrt yright)$$



                          Now, for any values $;x,y>1;$ , the left side above will be positive whereas the right side will be negative (can you see why?), thus we must restrict the possible natural values...Can you take it from here?






                          share|cite|improve this answer





















                          • sorry the question is edited. it was real.
                            – maveric
                            Nov 17 at 20:25










                          • can we say then 1?
                            – maveric
                            Nov 17 at 20:25










                          • The above solves completely the first, original question. Now that it has changed to "real" it at least can help you to realize that $;y;$ cannot be greater than $;1;$ and $;x;$ smaller than one, for example...
                            – DonAntonio
                            Nov 17 at 20:39















                          up vote
                          1
                          down vote













                          Observe that



                          $$x+y=sqrt x+sqrt yiff sqrt xleft(sqrt x-1right)=sqrt yleft(1-sqrt yright)$$



                          Now, for any values $;x,y>1;$ , the left side above will be positive whereas the right side will be negative (can you see why?), thus we must restrict the possible natural values...Can you take it from here?






                          share|cite|improve this answer





















                          • sorry the question is edited. it was real.
                            – maveric
                            Nov 17 at 20:25










                          • can we say then 1?
                            – maveric
                            Nov 17 at 20:25










                          • The above solves completely the first, original question. Now that it has changed to "real" it at least can help you to realize that $;y;$ cannot be greater than $;1;$ and $;x;$ smaller than one, for example...
                            – DonAntonio
                            Nov 17 at 20:39













                          up vote
                          1
                          down vote










                          up vote
                          1
                          down vote









                          Observe that



                          $$x+y=sqrt x+sqrt yiff sqrt xleft(sqrt x-1right)=sqrt yleft(1-sqrt yright)$$



                          Now, for any values $;x,y>1;$ , the left side above will be positive whereas the right side will be negative (can you see why?), thus we must restrict the possible natural values...Can you take it from here?






                          share|cite|improve this answer












                          Observe that



                          $$x+y=sqrt x+sqrt yiff sqrt xleft(sqrt x-1right)=sqrt yleft(1-sqrt yright)$$



                          Now, for any values $;x,y>1;$ , the left side above will be positive whereas the right side will be negative (can you see why?), thus we must restrict the possible natural values...Can you take it from here?







                          share|cite|improve this answer












                          share|cite|improve this answer



                          share|cite|improve this answer










                          answered Nov 17 at 20:24









                          DonAntonio

                          175k1491224




                          175k1491224












                          • sorry the question is edited. it was real.
                            – maveric
                            Nov 17 at 20:25










                          • can we say then 1?
                            – maveric
                            Nov 17 at 20:25










                          • The above solves completely the first, original question. Now that it has changed to "real" it at least can help you to realize that $;y;$ cannot be greater than $;1;$ and $;x;$ smaller than one, for example...
                            – DonAntonio
                            Nov 17 at 20:39


















                          • sorry the question is edited. it was real.
                            – maveric
                            Nov 17 at 20:25










                          • can we say then 1?
                            – maveric
                            Nov 17 at 20:25










                          • The above solves completely the first, original question. Now that it has changed to "real" it at least can help you to realize that $;y;$ cannot be greater than $;1;$ and $;x;$ smaller than one, for example...
                            – DonAntonio
                            Nov 17 at 20:39
















                          sorry the question is edited. it was real.
                          – maveric
                          Nov 17 at 20:25




                          sorry the question is edited. it was real.
                          – maveric
                          Nov 17 at 20:25












                          can we say then 1?
                          – maveric
                          Nov 17 at 20:25




                          can we say then 1?
                          – maveric
                          Nov 17 at 20:25












                          The above solves completely the first, original question. Now that it has changed to "real" it at least can help you to realize that $;y;$ cannot be greater than $;1;$ and $;x;$ smaller than one, for example...
                          – DonAntonio
                          Nov 17 at 20:39




                          The above solves completely the first, original question. Now that it has changed to "real" it at least can help you to realize that $;y;$ cannot be greater than $;1;$ and $;x;$ smaller than one, for example...
                          – DonAntonio
                          Nov 17 at 20:39










                          up vote
                          0
                          down vote













                          Let $f(x)=x-sqrt x$. The question becomes trying to find two real values $x,y$ so that $f(x)=-f(y)$, then maximising one of these values without regard for what happens to the other. After a bit of thinking analysing derivatives, it becomes clear that we want to minimise $f(x)$ so that $f(y)$, and hence $y$, can be maximised. So just set $f'(x)=0$ to find the local minimum value to be $-1/4$, and hence the maximum value of $y$ satisfies $f(y)=1/4$. This is a simple quadratic equation, solve, and you're done!






                          share|cite|improve this answer

























                            up vote
                            0
                            down vote













                            Let $f(x)=x-sqrt x$. The question becomes trying to find two real values $x,y$ so that $f(x)=-f(y)$, then maximising one of these values without regard for what happens to the other. After a bit of thinking analysing derivatives, it becomes clear that we want to minimise $f(x)$ so that $f(y)$, and hence $y$, can be maximised. So just set $f'(x)=0$ to find the local minimum value to be $-1/4$, and hence the maximum value of $y$ satisfies $f(y)=1/4$. This is a simple quadratic equation, solve, and you're done!






                            share|cite|improve this answer























                              up vote
                              0
                              down vote










                              up vote
                              0
                              down vote









                              Let $f(x)=x-sqrt x$. The question becomes trying to find two real values $x,y$ so that $f(x)=-f(y)$, then maximising one of these values without regard for what happens to the other. After a bit of thinking analysing derivatives, it becomes clear that we want to minimise $f(x)$ so that $f(y)$, and hence $y$, can be maximised. So just set $f'(x)=0$ to find the local minimum value to be $-1/4$, and hence the maximum value of $y$ satisfies $f(y)=1/4$. This is a simple quadratic equation, solve, and you're done!






                              share|cite|improve this answer












                              Let $f(x)=x-sqrt x$. The question becomes trying to find two real values $x,y$ so that $f(x)=-f(y)$, then maximising one of these values without regard for what happens to the other. After a bit of thinking analysing derivatives, it becomes clear that we want to minimise $f(x)$ so that $f(y)$, and hence $y$, can be maximised. So just set $f'(x)=0$ to find the local minimum value to be $-1/4$, and hence the maximum value of $y$ satisfies $f(y)=1/4$. This is a simple quadratic equation, solve, and you're done!







                              share|cite|improve this answer












                              share|cite|improve this answer



                              share|cite|improve this answer










                              answered Nov 18 at 6:22









                              YiFan

                              1,5091311




                              1,5091311






















                                  up vote
                                  0
                                  down vote













                                  There are some caveats (we need to assume the existence of a maximum for example) but Lagrange's multipliers method work well here to find the value of the maximum:



                                  Let $f(x,y)=x$ and $g(x,y)=x-sqrt{x}+y-sqrt{y}$. We want to find the maximum value of $f$ under constraint $g(x,y)=0$.



                                  We have $nabla f=langle 1,0 rangle$ and $nabla g = langle 1-frac{1}{2sqrt{x}},1-frac{1}{2sqrt{y}} rangle$. At a maximum, we have $nabla g=lambda nabla f$ for some real number $lambda$. This implies that $1-frac{1}{2sqrt{y}}=0$, that is $y=frac{1}{4}$.



                                  Using the constraint, we have $x-sqrt{x}-frac{1}{4}=0$ which is equivalent to $a^2-a-frac{1}{4}$ and $sqrt{x}=ageqslant 0$. We obtain $a=frac{1+sqrt{2}}{2}$, so $x=frac{(1+sqrt{2})^2}{4}=frac{3+2sqrt{2}}{4}$.





                                  (sloppy) proof of the existence of the global maximum value: The constraint $x+y=sqrt{x}+sqrt{y}$ implies that $xgeqslant 0$ and $ygeqslant 0$. Moreover, if $x$ or $y$ is large enough (x>R or y>R for some $R$ that I am too lazy to determine explicitly :p), $x+y>sqrt{x}+sqrt{y}$, so we can assume that $x,y in [0,R]$.



                                  Now, the function $f(x,y)=x$ is continuous and the set $S=left{ (x,y)in [0,R]^2; middle|; g(x,y)=0; right}$ is compact (closed and bounded in ${mathbb R}^2$) so, by the Extreme Value Theorem, $f$ attains its global extreme values on $S$. The global minimum value is obviously obtained at $(0,0)$, while the other one is located by Lagrange's method.






                                  share|cite|improve this answer

























                                    up vote
                                    0
                                    down vote













                                    There are some caveats (we need to assume the existence of a maximum for example) but Lagrange's multipliers method work well here to find the value of the maximum:



                                    Let $f(x,y)=x$ and $g(x,y)=x-sqrt{x}+y-sqrt{y}$. We want to find the maximum value of $f$ under constraint $g(x,y)=0$.



                                    We have $nabla f=langle 1,0 rangle$ and $nabla g = langle 1-frac{1}{2sqrt{x}},1-frac{1}{2sqrt{y}} rangle$. At a maximum, we have $nabla g=lambda nabla f$ for some real number $lambda$. This implies that $1-frac{1}{2sqrt{y}}=0$, that is $y=frac{1}{4}$.



                                    Using the constraint, we have $x-sqrt{x}-frac{1}{4}=0$ which is equivalent to $a^2-a-frac{1}{4}$ and $sqrt{x}=ageqslant 0$. We obtain $a=frac{1+sqrt{2}}{2}$, so $x=frac{(1+sqrt{2})^2}{4}=frac{3+2sqrt{2}}{4}$.





                                    (sloppy) proof of the existence of the global maximum value: The constraint $x+y=sqrt{x}+sqrt{y}$ implies that $xgeqslant 0$ and $ygeqslant 0$. Moreover, if $x$ or $y$ is large enough (x>R or y>R for some $R$ that I am too lazy to determine explicitly :p), $x+y>sqrt{x}+sqrt{y}$, so we can assume that $x,y in [0,R]$.



                                    Now, the function $f(x,y)=x$ is continuous and the set $S=left{ (x,y)in [0,R]^2; middle|; g(x,y)=0; right}$ is compact (closed and bounded in ${mathbb R}^2$) so, by the Extreme Value Theorem, $f$ attains its global extreme values on $S$. The global minimum value is obviously obtained at $(0,0)$, while the other one is located by Lagrange's method.






                                    share|cite|improve this answer























                                      up vote
                                      0
                                      down vote










                                      up vote
                                      0
                                      down vote









                                      There are some caveats (we need to assume the existence of a maximum for example) but Lagrange's multipliers method work well here to find the value of the maximum:



                                      Let $f(x,y)=x$ and $g(x,y)=x-sqrt{x}+y-sqrt{y}$. We want to find the maximum value of $f$ under constraint $g(x,y)=0$.



                                      We have $nabla f=langle 1,0 rangle$ and $nabla g = langle 1-frac{1}{2sqrt{x}},1-frac{1}{2sqrt{y}} rangle$. At a maximum, we have $nabla g=lambda nabla f$ for some real number $lambda$. This implies that $1-frac{1}{2sqrt{y}}=0$, that is $y=frac{1}{4}$.



                                      Using the constraint, we have $x-sqrt{x}-frac{1}{4}=0$ which is equivalent to $a^2-a-frac{1}{4}$ and $sqrt{x}=ageqslant 0$. We obtain $a=frac{1+sqrt{2}}{2}$, so $x=frac{(1+sqrt{2})^2}{4}=frac{3+2sqrt{2}}{4}$.





                                      (sloppy) proof of the existence of the global maximum value: The constraint $x+y=sqrt{x}+sqrt{y}$ implies that $xgeqslant 0$ and $ygeqslant 0$. Moreover, if $x$ or $y$ is large enough (x>R or y>R for some $R$ that I am too lazy to determine explicitly :p), $x+y>sqrt{x}+sqrt{y}$, so we can assume that $x,y in [0,R]$.



                                      Now, the function $f(x,y)=x$ is continuous and the set $S=left{ (x,y)in [0,R]^2; middle|; g(x,y)=0; right}$ is compact (closed and bounded in ${mathbb R}^2$) so, by the Extreme Value Theorem, $f$ attains its global extreme values on $S$. The global minimum value is obviously obtained at $(0,0)$, while the other one is located by Lagrange's method.






                                      share|cite|improve this answer












                                      There are some caveats (we need to assume the existence of a maximum for example) but Lagrange's multipliers method work well here to find the value of the maximum:



                                      Let $f(x,y)=x$ and $g(x,y)=x-sqrt{x}+y-sqrt{y}$. We want to find the maximum value of $f$ under constraint $g(x,y)=0$.



                                      We have $nabla f=langle 1,0 rangle$ and $nabla g = langle 1-frac{1}{2sqrt{x}},1-frac{1}{2sqrt{y}} rangle$. At a maximum, we have $nabla g=lambda nabla f$ for some real number $lambda$. This implies that $1-frac{1}{2sqrt{y}}=0$, that is $y=frac{1}{4}$.



                                      Using the constraint, we have $x-sqrt{x}-frac{1}{4}=0$ which is equivalent to $a^2-a-frac{1}{4}$ and $sqrt{x}=ageqslant 0$. We obtain $a=frac{1+sqrt{2}}{2}$, so $x=frac{(1+sqrt{2})^2}{4}=frac{3+2sqrt{2}}{4}$.





                                      (sloppy) proof of the existence of the global maximum value: The constraint $x+y=sqrt{x}+sqrt{y}$ implies that $xgeqslant 0$ and $ygeqslant 0$. Moreover, if $x$ or $y$ is large enough (x>R or y>R for some $R$ that I am too lazy to determine explicitly :p), $x+y>sqrt{x}+sqrt{y}$, so we can assume that $x,y in [0,R]$.



                                      Now, the function $f(x,y)=x$ is continuous and the set $S=left{ (x,y)in [0,R]^2; middle|; g(x,y)=0; right}$ is compact (closed and bounded in ${mathbb R}^2$) so, by the Extreme Value Theorem, $f$ attains its global extreme values on $S$. The global minimum value is obviously obtained at $(0,0)$, while the other one is located by Lagrange's method.







                                      share|cite|improve this answer












                                      share|cite|improve this answer



                                      share|cite|improve this answer










                                      answered 2 days ago









                                      Taladris

                                      4,62931832




                                      4,62931832






























                                           

                                          draft saved


                                          draft discarded



















































                                           


                                          draft saved


                                          draft discarded














                                          StackExchange.ready(
                                          function () {
                                          StackExchange.openid.initPostLogin('.new-post-login', 'https%3a%2f%2fmath.stackexchange.com%2fquestions%2f3002761%2fmaximum-value-of-x-when-equality-is-given%23new-answer', 'question_page');
                                          }
                                          );

                                          Post as a guest















                                          Required, but never shown





















































                                          Required, but never shown














                                          Required, but never shown












                                          Required, but never shown







                                          Required, but never shown

































                                          Required, but never shown














                                          Required, but never shown












                                          Required, but never shown







                                          Required, but never shown







                                          Popular posts from this blog

                                          Quarter-circle Tiles

                                          build a pushdown automaton that recognizes the reverse language of a given pushdown automaton?

                                          Mont Emei